Con Law: Multiple choice

Réussis tes devoirs et examens dès maintenant avec Quizwiz!

Chief Justice Taft wrote in the Child Labor Tax Case (1922): "Taxes are occasionally imposed in the discretion of the Legislature on proper subjects with the primary motive of obtaining revenue from them and with the incidental motive of discouraging them by making their continuance onerous. They do not lose their character as taxes because of the incidental motive. But there comes a time in the extension of the penalizing features of the so-called tax when it loses its character as such and becomes a mere penalty, with the characteristics of regulation and punishment." Which of the following statements is true? A In National Federation of Independent Businesses v. Sebelius (2012), the Court did not consider whether the purported tax at issue had the primary motive of raising revenue. B Kahriger v. United States (1953) limited the effect of the Child Labor Tax Case (1922) by holding that Congress can use so-called taxes to regulate subjects so long as the regulations are not extraneous to any tax needs. C Chief Justice Taft was concerned that Congress would use its power of taxation to regulate matters that it could not regulate under the Commerce Clause. D All of the above.

D All of the above. Correct. All of the statements are true.

Which of the following factors does the Supreme Court consider relevant to determining whether a federal statute regulating intrastate activity is within Congress's Commerce Power? (1) The existence or absence of a jurisdictional element.(2) The existence or absence of findings of fact by Congress.(3) Whether the intrastate activity is economic in nature.(4) Express indications of state government acquiescence to federal regulation. A (1) and (4) only B (2) and (3) only C (1), (2), and (3) only D (1), (2), (3), and (4)

C (1), (2), and (3) only Correct. In cases such as United States v. Lopez (1995), United States v. Morrison (2000), and Gonzales v. Raich (2005), the Supreme Court considered (1) the existence or absence of a jurisdictional element, (2) the existence or absence of findings of fact by Congress, and (3) whether the intrastate activity is economic in nature. The Court did not consider express indications of state government acquiescence to federal regulation, even though Justice Souter thought it was relevant in his dissent in Morrison.

The New York Times has just published an article reporting that President Smith is under investigation for violating campaign finance rules while running for his first term of office. A White House lawyer tells President Smith, "Don't worry, Mr. President. We can fight this." Which defense is likely to be successful? A An assertion of Executive Immunity for official acts. B An assertion of Executive Privilege for official communications. C An assertion that the Separation of Powers prevents intra-branch prosecutions. D None of the above.

D None of the above. Correct. None of the listed arguments is likely to be a successful defense.

Which of the following assertions accurately state the current understanding of the application of provisions of the Bill of Rights to the states? 1. The provisions of the first eight amendments to the Constitution apply to the states. 2. The Due Process Clause of the Fifth Amendment requires application of the privilege against self-incrimination to the states. 3. Most, but not all, of the provisions of the Bill of Rights apply to the states through the Due Process Clause of the Fourteenth Amendment. 4. Most, but not all, of the provisions of the Bill of Rights apply to the states through the Privileges or Immunities Clause of the Fourteenth Amendment. A 1, 2, and 3 B 1 and 4 C 1 and 3 D 2 and 3

D 2 and 3 Correct. For the reasons noted above, assertions 2 and 3 are the only accurate statements of current law.D 2 and 3Correct. For the reasons noted above, assertions 2 and 3 are the only accurate statements of current law.

Based on what the Supreme Court said in Morrison v. Olson (1988), which of the following statements is false? A Congress can empower federal judges to appoint federal prosecutors. B Federal prosecutors are not necessarily principal officers of the United States. C Congress can authorize federal judges to engage in non-judicial ministerial tasks. D A corollary of the President's duty to see that the laws are faithfully executed is that the President must have complete control over all federal prosecutions.

D A corollary of the President's duty to see that the laws are faithfully executed is that the President must have complete control over all federal prosecutions. Correct. This statement is false. In Morrison v. Olson (1988), the Supreme Court upheld a statute that authorized prosecution decisions to be made independently of the President.

On which of the following reasons did the Court rely in Dred Scott v. Sandford in denying relief to Scott? 1. Scott and the defendant were from the same state, and thus there was no diversity of citizenship. 2. Scott was not a citizen of the United States and thus there was no subject matter jurisdiction over the suit. 3. Scott's time in the free territory of Wisconsin did not make him a free citizen because Congress lacked power to abolish slavery in territories. 4. No state deprived Scott of life, liberty, or property within the meaning of the Fifth Amendment. 5. Scott's time in the free state of Illinois did not make him a free citizen because under Missouri law he was still a slave. A 2, 3, and 5 B 1, 3, and 4 C 2, 4, and 5 D 1, 3, 4, and 5

A 2, 3, and 5 Correct. The Court relied on reasons 2, 3, and 5. Reason 2 is correct because the Court reasoned that Scott, as a black man sold as a slave, was not a citizen within the meaning of the term in Article III (among other provisions), and accordingly could not invoke subject matter jurisdiction under the theory of diversity of citizenship. Reason 3 is correct because the Court held that the Missouri Compromise deprived slave owners of their property—that is, their slaves—without due process of law in violation of the Fifth Amendment. Finally, reason 5 is correct because the Court held that Scott's status "depended on the laws of Missouri," and the Missouri Supreme Court had concluded that Scott was not a free man.

On which of the following reasons did the Court in Barron v. Baltimore rely in concluding that the City of Baltimore's action was constitutional? 1. Because the states have their own constitutions, their actions are not limited by the federal Constitution. 2. Because the Constitution was designed to identify and limit the powers of the federal government, it does not limit the power of the states to interfere with individual rights. 3. Because the original Constitution includes a separate section identifying specific limits on the powers of the states, the provisions of the Bill of Rights should be understood to apply only to federal action absent an explicit statement that they also apply to state action. 4. Because during the debates over ratification of the Constitution many skeptics expressed fears over the possibility that the federal government would use its powers to interfere with individual rights, and demanded the adoption of a bill of rights to address those concerns, the first eight amendments should be understood to limit only the power of the federal government. 5. Because the City of Baltimore did not actually assert the power of eminent domain over Barron's wharf, its action could not have violated the Fifth Amendment, which applies only when property is "taken for public use." A 3 and 4 B 2, 3, and 4 C 1 and 2 D 4 and 5

A 3 and 4 Correct. Assertions 3 and 4 are the only correct responses. Assertion 3 is correct because the Court reasoned that the original Constitution, in Article I, Sections 9 and 10, specifically distinguished between limitations on the federal and state governments, and thus that the Constitution applies to state action only when it specifically so states. Assertion 4 is correct because the Court reasoned that "it is universally understood" that the Bill of Rights responded to demands for "security against the apprehended encroachments of the general government—not against those of the local governments."

The United States government is prosecuting an accused terrorist following a horrific bombing. The accused has requested the court to issue a subpoena to the President of the United States requiring him to turn over a memorandum written by the President's National Security Advisor. The accused has probable cause to believe that the memorandum will implicate someone else as the perpetrator of the bombing, thus giving him a defense to the crime. The President has refused to turn over the memorandum, citing executive privilege. Which of the following statements is true? A Absent a claimed need to protect military, diplomatic, or sensitive national security secrets, the court could order the President to turn over the memorandum for an in camera review to determine whether the memorandum contains relevant, non-privileged evidence. B An absolute executive privilege completely shields all communications between high Government officials and those who advise and assist them. C Executive Privilege does not apply in criminal cases. D Executive Privilege does not apply unless the President is a defendant in a criminal case.

A Absent a claimed need to protect military, diplomatic, or sensitive national security secrets, the court could order the President to turn over the memorandum for an in camera review to determine whether the memorandum contains relevant, non-privileged evidence. Correct. The Supreme Court held that a court may use an in camera review procedure in cases that do not involve claims of the need to protect "military, diplomatic, or sensitive national security secrets." United States v. Nixon (1974).

Which is an accurate statement about the Supreme Court's decision in District of Columbia v. Heller? A All of the Justices agreed that it was proper to consider the original meaning of the Second Amendment in deciding the case and proper to rely on historical materials to make claims about the original meaning. B In their separate opinions, Justice Stevens and Justice Breyer rejected originalist methodology. C The Court held that the right secured by the Second Amendment is unlimited. D Justice Stevens and Justice Scalia drew essentially the same conclusion in looking at state constitutions from the Founding Era that had provisions similar to the Second Amendment.

A All of the Justices agreed that it was proper to consider the original meaning of the Second Amendment in deciding the case and proper to rely on historical materials to make claims about the original meaning. Correct. All of the Justices joined either the majority or the dissent, and both opinions cited historical sources to support their claims about the original meaning.

You work on the legal staff of a United States senator. The senator is upset because the President has just announced that the President has withdrawn the United States from an international treaty regarding global warming. The senator worked for many years to obtain Senate approval for the treaty, and is disappointed in the President's action. What advice should you give the senator about contesting the President's action? A As a practical matter, the only recourse is politics, because the courts will not interfere. B The treaty remains in force. The only way to exit from the treaty is to have Congress pass a law that supersedes the previous international agreement. C The President's action is not effective until approved by a vote of two thirds of the Senate. D The President's actions directly violate the principles of United States v. Curtiss-Wright Export (1936).

A As a practical matter, the only recourse is politics, because the courts will not interfere. Correct. As a practical matter, the President can exercise some foreign affairs powers not expressly granted in the Constitution, because the federal courts will decline to review the President's actions under the political question doctrine or other doctrines. For example, the Supreme Court declined in Goldwater v. Carter (1979) to decide whether the President had the power to rescind treaty obligations, effectively allowing the President to rescind a treaty.

Congress wants to create a new agency to enforce its recent health care legislation. Because the majority in the House and Senate are of a different party from the President, they also want to limit the President's power over the new agency. What power does Congress have to limit the President's control? A Congress may restrict the President's power to remove the members of a bi-partisan, multimember commission if their duties are not properly considered executive in nature. B Congress may appoint the head of the new agency by joint resolution. C Congress can limit the President's power to remove the head of the new agency if agency independence is consistent with the need for good government. D Congress can reserve to itself the power to remove the head of the new agency if it disagrees with the agency's actions.

A Congress may restrict the President's power to remove the members of a bi-partisan, multimember commission if their duties are not properly considered executive in nature. Correct. This statement is true. Congress may restrict the President's power to remove the heads of a multimember commission if the commission is bi-partisan and does not meaningfully exercise executive authority. The Court upheld such an arrangement in Humphrey's Executor v. United States (1935) and affirmed that holding in Seila Law v. Consumer Financial Protection Bureau (2020).

Which conclusion can be drawn from the Supreme Court's decision in Ex Parte McCardle (1869)? A If Congress is concerned about the outcome of a case pending before the Supreme Court, Congress may have the power under the Exceptions Clause in Article III, section 2, clause 2, to prevent the Supreme Court from deciding the case by stripping the court of appellate jurisdiction. B Congress cannot make exceptions to the original jurisdiction of the Supreme Court but it can expand that jurisdiction. C Congress can make appellate review by the Supreme Court of certain classes of cases discretionary but the Supreme Court never has mandatory appellate jurisdiction. D Congress can completely deprive the Supreme Court of jurisdiction over a particular federal question.

A If Congress is concerned about the outcome of a case pending before the Supreme Court, Congress may have the power under the Exceptions Clause in Article III, section 2, clause 2, to prevent the Supreme Court from deciding the case by stripping the court of appellate jurisdiction. Correct. This is exactly what happened in Ex Parte McCardle (1869). Congress passed legislation creating an exception to the Supreme Court's appellate jurisdiction to prevent the Supreme Court from deciding a case.

The Illinois state legislature has passed a law barring any person convicted of a felony from being listed as a candidate on any election ballot within the state. The former governor, who was convicted of a felony several years ago, is now running for the U.S. Senate. Despite his criminal past, most polls indicate that the former governor would have a very strong chance of being elected if his name appears on the ballot. The governor seeks to challenge the Illinois law. Which of the following statements is correct? A Illinois may not bar felons from being listed as candidates on election ballots in U.S. Senate elections. B Congress may bar felons from being listed as candidates on election ballots in U.S. Senate elections. C Illinois can bar felons from being listed as candidates on election ballots but cannot prevent them from being elected to the U.S. Senate by write-in ballot. D The former governor lacks standing to challenge the law.

A Illinois may not bar felons from being listed as candidates on election ballots in U.S. Senate elections. Correct. Article I sets forth the only qualifications for members of Congress, and the states may not attempt to impose additional qualifications by limiting the ability of candidates to have their names appear on general ballots. U.S. Term Limits, Inc. v. Thornton (1995).

Under the Dormant Commerce Clause Doctrine, a state law may impose a burden on interstate commerce so long as the burden is not excessive in relation to legitimate local interests. Which of the following would most likely not be a legitimate local interest for the purpose of this test? A Preserving industries hampered by high local labor costs. B Discouraging the formation of violent youth gangs. C Preventing anthropogenic (i.e., human caused) global climate change. D Maintaining the purity of water in rivers that extend into other states.

A Preserving industries hampered by high local labor costs. Correct. For the purpose of the Dormant Commerce Clause Doctrine, legitimate state interests include protecting health and safety and conserving natural resources, but they do not include protecting local businesses from competition. Dean Milk v. City of Madison (1951). Preserving local industries hampered by high local labor costs sounds like protecting local businesses from non-local competition.

Congress believes that the rates charged by railroads for interstate shipments of goods should be regulated. But Congress is reluctant to regulate the rates charged for these shipments by statute for two reasons. First, any statute enacted would have to be very complex because of the number of different kinds of shipments, the variations in local conditions, and a variety of other factors. Second, Congress is worried about having to update its legislation to account for changes in prices, development of new technology, and similar matters. Which one of the following options is open to Congress? A Set up an executive agency, and give the executive agency discretion to set the rates charged by railroads for interstate shipments. B Enact legislation requiring each state to regulate designated shipments according to specified principles. C Give a governmental official within the legislative branch discretion to set the rates charged by railroads for interstate shipments. D Allow the railroads to set rates that would go into effect 90 days after their announcement, subject to a veto by a resolution of the majority of either house of Congress.

A Set up an executive agency, and give the executive agency discretion to set the rates charged by railroads for interstate shipments. Correct. Congress may give the Executive Branch discretion to promulgate rules and regulations so long as Congress guides the discretion with some "intelligible principle." Whitman v. American Trucking (2007). In fact, for many years, the Interstate Commerce Commission did regulate railroad rates for interstate shipments.

Congress has passed a law outlawing the use of marijuana. Although the Supreme Court has upheld the law as constitutional, the President has announced that the Justice Department will not enforce the law in states where state law does not prohibit the use of marijuana. Which of the following statements is correct? A State law has no bearing on whether the President's actions are constitutional. B The President's actions are constitutional because they fall within the first category of executive actions identified by Justice Jackson in his separate opinion in Youngstown Sheet & Tube v. Sawyer (1952). C The President's duty to ensure that the laws are faithfully executed affords no discretion in deciding whether or how to prosecute crimes. D Congress may respond to the President's actions by creating an independent federal agency, over which the President has no control, and then giving that agency authority to prosecute marijuana offenses.

A State law has no bearing on whether the President's actions are constitutional. Correct. The President is responsible for faithfully executing federal law. Because federal law is supreme over state law under the Supremacy Clause, state law has no bearing on the President's action.

Which of the following statements regarding Congress's powers is false? A The Constitution, like the Articles of Confederation, excludes incidental or implied powers, and requires every power granted to Congress to be expressly and minutely described. B Congress may seek to achieve any end that is legitimate under its express powers as long as its chosen means are adapted to achieving that end. C The Necessary and Proper Clause ensures that Congress has the subsidiary power to carry out the great powers listed in Article I, section 8, clauses 1-17. D Some of the powers listed in Article I, section 8 are exclusively federal powers but some are not.

A The Constitution, like the Articles of Confederation, excludes incidental or implied powers, and requires every power granted to Congress to be expressly and minutely described. Correct. This statement is false. It is an altered version of a statement reaching precisely the opposite conclusion in McCulloch v. Maryland (1819). The Articles of Confederation contained a clause saying that Congress had only those powers "expressly" delegated by the states. The Constitution contains no such phrase. Chief Justice Marshall characterized powers that are not "minutely described," such as the power to charter corporations, as "implied powers." Some critics have suggested this terminology is confusing because the power is granted by the Necessary and Proper Clause. But the statement as written is still false because, as this example shows, the Constitution grants powers that are not "expressly and minutely described."

Agents of a hostile nation succeeded in mining and sinking a U.S. Navy supply vessel. Congress quickly passed a joint resolution authorizing the President of the United States to use military force against this nation. Which of the following statements is true? A The President can detain captured forces of the hostile nation provided that the President acts in accordance with federal statutes and the international law of war. B The President can detain captured forces of the hostile nation and, because of the joint resolution's authorization to use military force, is not limited by the laws of war. C Notwithstanding the joint resolution, the president cannot use military force absent permission from the UN Security Council. D The President's use of force is a political question that cannot be reviewed by the federal courts.

A The President can detain captured forces of the hostile nation provided that the President acts in accordance with federal statutes and the international law of war. Correct. If federal legislation grants the President authority to use military force, but does not specify the details, the President has only the powers granted by the Constitution, by other statutes, and by the laws of war. Exercising these powers, the President may detain enemy combatants, if they are accorded certain requirements imposed by the Due Process Clause. Hamdi v. United States (2004).

The Montana legislature enacted a statute that provides that, before performing an abortion in the first twenty weeks of pregnancy, doctors must show women seeking to obtain such an abortion a five-minute video with footage of an actual fetus and the fetus's beating heart. The statute further provides that, after viewing the video, women must wait 48 hours before obtaining an abortion. Peggy Waters, a woman in her twelfth week of pregnancy who wishes to obtain an abortion, has filed a lawsuit contending that the statute is unconstitutional and seeking an injunction preventing the statute's enforcement against her. She has alleged that she lives seven hours from the nearest doctor who performs abortions and will be forced to miss three days of work because of the statute's requirements.How should the court rule? A The court should invalidate the waiting-period requirement as applied to Waters because it imposes an undue burden on her right to obtain an abortion, but it should not disturb the video requirement because it does not impose an undue burden on her right to obtain an abortion. B The court should invalidate the statute on its face because the Due Process Clause prohibits the state from regulating abortion in the first trimester of a pregnancy. C The court should uphold the statute as applied to Waters because the video and waiting-period requirements are sensible measures to ensure that the decision whether to obtain an abortion is well informed. D The court should dismiss the suit because Waters has not alleged that she sought an abortion and was denied treatment as a result of the statute.

A The court should invalidate the waiting-period requirement as applied to Waters because it imposes an undue burden on her right to obtain an abortion, but it should not disturb the video requirement because it does not impose an undue burden on her right to obtain an abortion. Correct. The undue burden standard applies to regulations of abortion before viability. Although the exact content of the undue burden test is unclear, the Court in Casey made clear that a waiting-period requirement could, in certain circumstances such as those present here, impose an undue burden on the right to obtain an abortion. In contrast, the video requirement is very similar to the requirement upheld in Casey requiring doctors to inform women of the probable gestational age of the unborn child.

Marvin Douglas worked for the Virginia Department of Transportation. As part of his compensation, the state provided a life insurance policy. When he was 50 years old, Douglas was diagnosed with terminal cancer. Douglas did not want to undergo treatment, which likely would have left him weak and uncomfortable, and he instead committed suicide. When his wife, the beneficiary of his state-provided life insurance policy, filed a claim under the policy, the state denied the claim, explaining that, under the legislatively adopted rules for life insurance policies for state employees, suicide invalidates the policy. Douglas's wife has filed suit contending that the exclusion for suicide violates the Due Process Clause of the Fourteenth Amendment.How should the court rule? A The court should uphold the rule excluding suicide from coverage because the rule is rationally related to the legitimate government interest in the preservation of human life. B The court should uphold the rule excluding suicide from coverage because, although the Due Process Clause protects a fundamental right to end one's life with dignity, the rule is narrowly tailored to advance the state's compelling interest in protecting human life. C The court should invalidate the rule excluding suicide from coverage because the Due Process Clause protects a fundamental right to end one's life with dignity, and the rule does not further a compelling government interest. D The court should invalidate the rule excluding suicide from coverage because it burdens persons already grieving over the loss of a loved one.

A The court should uphold the rule excluding suicide from coverage because the rule is rationally related to the legitimate government interest in the preservation of human life. Correct. The Due Process Clause does not protect a fundamental right to commit suicide; because the policy accordingly is subject only to rational-basis review; and because the policy, like the law at issue in Glucksberg, is rationally related to the state's legitimate interest in protecting human life. The Court in Glucksberg, in concluding that the Due Process Clause does not protect a fundamental right to assistance in committing suicide, strongly suggested that the Clause also does not protect the anterior right to commit suicide. For example, the Court noted that the question before it was "whether the 'liberty' specially protected by the Due Process Clause includes a right to commit suicide which itself includes a right to assistance in doing so," and then noted that there was a "consistent and almost universal tradition that has long rejected the asserted right."

Congress passed a law outlawing the use of marijuana, even for medical purposes. A litigant in California challenged the constitutionality of the law, but the U.S. Supreme Court upheld the statute. Subsequently, the Oregon legislature enacted a statute permitting the use of medical marijuana within its borders and declaring that the federal law was unconstitutional. Which of the following statements is true? A The federal government may enforce the federal law against persons using marijuana for medical purposes in Oregon. B The Oregon legislature's determination that the federal law is unconstitutional is entitled to "substantial deference" because all state legislators take an oath to uphold the Constitution.Incorrect. Although state legislators take an oath to uphold the Constitution, the doctrine of "judicial supremacy" says that the federal courts ultimately determine the meaning of the Constitution. C Congress may repeal the Oregon statute.Incorrect. Congress cannot "repeal" state laws. Congress can preempt state laws with federal law but the preemption continues only so long as the federal law exists. D The Oregon governor can order that no state or federal officials enforce the law in Oregon.Incorrect. A state governor may have power to order state officials not to enforce a federal law but the governor has no power to order federal officials to take or refrain from taking any action.

A The federal government may enforce the federal law against persons using marijuana for medical purposes in Oregon. Correct. Under the doctrine of "judicial supremacy," the Supreme Court's determination of the constitutionality of the federal law is conclusive. Cooper v. Aaron (1958); Dickerson v. United States (2000). The Oregon legislature's declaration that the law is unconstitutional does not prevent enforcement of the law.

Joining the popular "locally grown food" movement, the Vermont legislature recently passed a law requiring 90% of the food and beverages that Vermont public schools serve to students as school lunches to be produced in Vermont. Farmers in New Hampshire, who grow agricultural products that Vermont public schools have regularly purchased, have filed a lawsuit challenging the law. Which of the following statements is correct? A The law does not violate the Dormant Commerce Clause Doctrine. B The law impermissibly discriminates against interstate commerce. C A court can uphold the law only if it finds that buying locally grown food serves an important state interest (e.g., preserving the environment) and that Vermont cannot address this interest in a less discriminatory manner. D The law is unconstitutional as currently written, but the law would be constitutional if it required public schools to purchase food and beverages produced within 100 miles of their physical locations.

A The law does not violate the Dormant Commerce Clause Doctrine. Correct. Under the Market-Participant Exception to the Dormant Commerce Clause Doctrine, a state may discriminate against interstate commerce when buying or selling goods or services in the market (as opposed to regulating the market). South-Central Timber Development, Inc. v. Wunnicke (1982).

A Louisiana statute requires all bus drivers to obtain a Louisiana bus operator's license before driving a bus in Louisiana. Anyone can obtain such a license by paying a nominal fee and passing a written test and a road test. The written test is difficult and even experienced truckers must study to pass it. All other states allow anyone with a valid bus operator's license issued by any state to drive a bus on their roads. Louisiana roads and traffic laws are very similar to those of its neighboring states. A nationwide bus company has challenged the Louisiana law. Which argument against the law is the strongest? A The law places an excessive burden on interstate commerce in violation of the Dormant Commerce Clause Doctrine. B The law discriminates against interstate commerce in violation of the Dormant Commerce Clause Doctrine. C The law violates the Privileges and Immunities Clause. D The law is preempted by federal law because Congress has occupied the field of interstate transportation.

A The law places an excessive burden on interstate commerce in violation of the Dormant Commerce Clause Doctrine. Correct. The Dormant Commerce Clause Doctrine prohibits a state law from imposing a burden on interstate commerce that is excessive in relation to legitimate local interests. South Carolina Department of Transportation v. Barnwell Brothers (1938); Southern Pacific v. Arizona (1945); Kassel v. Consolidated Freightways Corp. (1981). The burden of this law on interstate commerce is substantial because it prevents bus drivers with licenses from other states from driving through Louisiana without applying for and obtaining a license. Even if the fee is nominal and tests are passable, imagine the burden on interstate busing if all states adopted such laws. The state has a legitimate local interest in traffic safety but requiring a Louisiana license to protect this interest seems excessive because no other state has a similar law and Louisiana's traffic laws and roads are not different from those in neighboring states.

The President of the United States engaged in an inappropriate sexual relationship with an intern and then testified falsely about the incident under oath. Some members of Congress have argued that the President should be impeached by the House and tried by the Senate for committing perjury. Other members of Congress have sought a compromise. They have suggested passing a joint resolution reprimanding the President and fining him $20,000 for his misconduct. Which of the following statements is true? A The proposed compromise would be distinguishable from the law at issue in Nixon v. Administrator of General Services (1977) because there would be a legislative intent to punish the President. B A joint resolution cannot have the force of law and therefore cannot be unconstitutional. C Any law that applies to just one person is a bill of attainder and is unconstitutional. D If the President is impeached, and the Senate convicts him, the Senate could impose a reprimand and a fine as a punishment.

A The proposed compromise would be distinguishable from the law at issue in Nixon v. Administrator of General Services (1977) because there would be a legislative intent to punish the President. Correct. In the hypothetical, Congress would be enacting the joint resolution for the purpose of punishing the President—by fining him and reprimanding him—for perjury. In contrast, the Supreme Court upheld the law in Nixon v. Administrator of General Services (1977) because it concluded that Congress had enacted the law for non-punitive purposes.

After unsuccessfully fighting worsening air pollution for many years, Los Angeles enacted an ordinance prohibiting the use of privately owned gasoline-powered vehicles on city streets within a 25 square-mile sector of the city except on Sundays. Although Los Angeles has increased the available modes of public transportation, many residents who live within the restricted area are outraged because they cannot use their cars to go to work or for many personal purposes. Those who have sold their cars have received disappointingly low prices because of the sudden glut of used vehicles in the area and the diminished demand for cars. Others have sued the city, claiming that the ordinance "goes too far" and amounts to a regulatory taking of their vehicles. Which of the following arguments should Los Angeles make in response? A The regulation is not a taking based on the reasoning in Andrus v. Allard (1979) because the vehicle owners can still use their cars for some purposes. B The regulation is not a taking because the city has an absolute right to exclude vehicles from city streets for any reason. C The regulation is not a taking because the city can ban nuisances, and vehicles are nuisances. D Even if the regulation is a taking, the residents have already received just compensation in the form of increased public transportation.

A The regulation is not a taking based on the reasoning in Andrus v. Allard (1979) because the vehicle owners can still use their cars for some purposes. Correct. In Andrus v. Allard (1979), the Supreme Court held that a federal law that deprived owners of many but not all of their rights with respect to property (i.e., eagle feathers) was not a regulatory taking. Los Angeles could use the same argument here, contending the vehicle owners can still drive their cars outside of the downtown area, use their cars on Sunday, sell their cars, pledge them as collateral, etc.

After a series of setbacks in a military conflict between the United States and foreign fighters in the Middle East, Congress lost confidence in the President's ability to make and implement a strategy that would lead to victory. Congress passed a statute requiring the President to appoint an "Independent General," subject to approval by the Senate, who would command all U.S. forces in the conflict. The statute specified that the President could not interfere with the Independent General's decisions and could fire the Independent General only for good cause, which the statute defined narrowly as misconduct, neglect, or incapacitation but not mere disagreement over policy. What is the best argument that the statute is unconstitutional? A The statute impermissibly limits the President's constitutional authority to oversee the command structure of the Armed Forces. B The Constitution does not permit a statute to condition a presidential appointment on approval by the Senate. C Although Congress has the power to declare war, Congress must leave all aspects of the conduct of a war to the President. D The statute is unconstitutional because the definition of good cause does not include "malfeasance."

A The statute impermissibly limits the President's constitutional authority to oversee the command structure of the Armed Forces. Correct. This is the best argument. Article II makes the President the Commander-in-Chief of the Armed Forces. The hypothetical statute in this problem prevents the President from serving in this capacity by allowing a commander of some of the Armed Forces to operate independently. In addition, the Supreme Court has made clear that Congress may limit the President's power to remove an officer only in very limited circumstances that are not present here. Seila Law v. Consumer Financial Protection Bureau (2020).

Congress has passed two laws, the License Act and the Truth in Applications Act. The License Act requires any person driving an interstate truck route to have a federal trucking license. The Truth in Applications Act makes it a crime to make a knowingly false statement when applying for a federal trucking license. Smith intentionally lied on his federal trucking license application, falsely asserting that he did not have previous drunk-driving convictions. He was prosecuted and convicted of violating the Truth in Applications Act. Does the Necessary and Proper Clause give Congress the power to pass these Acts? A Yes, the Necessary and Proper Clause gives Congress the power to pass both Acts. B The Necessary and Proper Clause gives Congress the power to pass the License Act but not the Truth in Applications Act. C The Necessary and Proper Clause gives Congress the power to pass the Truth in Applications Act but not the License Act. D No, Congress does not have power to pass either Act.

A Yes, the Necessary and Proper Clause gives Congress the power to pass both Acts. Correct. The License Act is necessary and proper for carrying out Congress's power to regulate interstate commerce. Courts are unlikely to hold that the link between the Truth in Applications Act and the power to regulate interstate commerce is too attenuated because of the importance of having valid information when issuing licenses. The Supreme Court has rejected the argument that "the Necessary and Proper Clause permits no more than a single step between an enumerated power and an act of Congress." United States v. Comstock (2010).

Which of the following statements concerning the grant of legislative power to Congress in Article I and the grant of executive power to the President in Article II are true? (1) Article II does not include anything like the Necessary and Proper Clause found in Article I, section 8, clause 18. (2) Article I and Article II both contain express prohibitions on the exercise of federal power. (3) Unlike Article I, which purports to vest in Congress only the legislative powers "herein granted," Article II vests in the President the "executive Power" without qualification. A (1) only B (1) and (3) only C (2) and (3) only D (1), (2), and (3)

B (1) and (3) only Correct. Statements (1) and (3) are true. Statement (1) is true because the Necessary and Proper Clause authorizes Congress to make any laws necessary for carrying out its enumerated powers, a provision that the Supreme Court has interpreted to grant implied powers to Congress. Article II contains no comparable provision. Statement (3) is correct because the text of the first sentence of Article I vests in Congress the powers "herein granted" while the text of the first sentence of Article II vests in the President "the executive power" without qualification.

In South Dakota v. Dole (1987), the Supreme Court recognized that Congress could use conditional spending to influence a state to take a particular action. The Court, however, identified several requirements for the use of conditional spending to be constitutional. Which of the following are not requirements? (1) The exercise of the spending power must be in pursuit of "the general welfare" (2) Conditions imposed upon the states' receipt of federal funds must be limited in duration (3) The conditions must be related to the federal interest in the spending program (4) The conditions must not violate any independent constitutional bar (5) The financial inducement must not entice a state to act in an area beyond the enumerated powers of Congress A (1) only B (2) and (5) C (3) and (4) D (5) only

B (2) and (5) Correct. The requirements in (2) and (5) are misstated. The actual requirements are (2) conditions imposed upon the states' receipt of federal funds must be unambiguous; and (5) the financial inducement offered by Congress must not be so coercive as to pass the point at which "pressure turns into compulsion." South Dakota v. Dole (1987).

Which of these hypothetical laws is most likely to violate the Penalty Doctrine concerning Congress's taxation power? A A punitive tax that seeks to regulate conduct that Congress cannot reach under the Commerce Clause and that raises exorbitant sums from affected businesses. B A punitive tax that seeks to regulate conduct that Congress cannot reach under the Commerce Clause and that raises no revenue. C A punitive tax that is not linked to any kind of interstate activity. D A punitive tax that makes no allowance for taxpayers unable to pay.

B A punitive tax that seeks to regulate conduct that Congress cannot reach under the Commerce Clause and that raises no revenue. Correct. As initially developed in the Child Labor Tax Case (1922), the Penalty Doctrine said that Congress could not use tax laws to regulate conduct not otherwise within the reach of Congress's Commerce or other affirmative powers. In United States v. Kahriger (1953), however, the Supreme Court held that the Penalty Doctrine applied only to tax laws "extraneous to any tax need"—that is, effectively, only to tax laws that do not raise any revenue.

What is the theory of the "unitary Executive"? A Judicial review of executive power is restricted to the express limits found in Article II. B All of the executive power, not just some of it, is vested in the President of the United States. C The President can seek written advice from heads of departments but must personally make "essential" executive decisions. D The so-called "independent agencies" are unconstitutional because they report to Congress rather than to the President.

B All of the executive power, not just some of it, is vested in the President of the United States. Correct. This is a paraphrase of Justice Scalia's classic exposition of the theory of the unitary executive in his dissent in Morrison v. Olson (1988). The theory is that the President must have ultimate control over any exercise of purely executive power.

How does the structure of the Constitution reflect the Framers' vision that there should be three branches of government: the legislative, the executive, and the judicial? A By precluding any involvement of one branch in the exercise of another branch's powers or authority. B By separately defining each of the branches of government and their powers. C By creating a hierarchy, with the Supreme Court being above Congress, and Congress being above the President. D By establishing federalism, and sharing of power between the states governments and the federal government.

B By separately defining each of the branches of government and their powers. Correct. The Constitution defines and lists the powers of the three branches of government in Articles I, II, and III of the Constitution. This organization was a change from the Articles of Confederation, which did not establish separate branches.

Congress sometimes takes actions that test the separation of powers between the legislative and executive branches of government. Which of the following statements regarding legislative power is correct? A Congress can delegate legislative power to independent administrative agencies which can issue binding regulations. B Congress can enact statutes that give Executive Branch agencies discretion, guided by intelligible principles, to promulgate binding regulations, even if this means that the Executive Branch can make significant policy decisions. C The House and Senate can, in effect, make laws without involving the President through the process of passing joint resolutions. D The federal courts cannot appoint an official who exercises executive powers.

B Congress can enact statutes that give Executive Branch agencies discretion, guided by intelligible principles, to promulgate binding regulations, even if this means that the Executive Branch can make significant policy decisions. Correct. Congress may give the executive branch discretion to promulgate rules and regulations so long as Congress guides the discretion with some "intelligible principle." Whitman v. American Trucking (2007).

Prior to 1937, in cases like Carter v. Carter Coal Co. (1936), what did the Supreme Court say was beyond Congress's power to regulate under the Commerce Clause? A Commerce that concerns more than one state. B Intrastate economic activity that, by itself, has only an indirect effect on interstate commerce. C The channels of commerce, such as roads and navigable waters. D The instrumentalities of commerce, such as vessels, trains, and trucks.

B Intrastate economic activity that, by itself, has only an indirect effect on interstate commerce. Correct. In Carter v. Carter Coal Co. (1936), the Supreme Court held that Congress could not regulate the wages and hours of coal miners because a coal miner's production, at most, had an indirect effect on interstate commerce. The Supreme Court recognized that Congress could regulate intrastate economic activities that affect interstate commerce in cases such as United States v. Darby (1941), which overruled Carter Coal.

The City of Monroe enacted an ordinance prohibiting anyone from selling pasteurized milk within the city limits unless the milk was pasteurized within 5 miles of the city center. The Supreme Court struck down this law under the Dormant Commerce Clause doctrine on grounds that the law discriminated against interstate commerce. In response to intense lobbying by local politicians, Congress responded to the Supreme Court's decision by enacting a statute authorizing cities to enact ordinances of the kind formerly passed by Monroe and struck down by the Supreme Court. Which of the following statements is correct? A The statute passed by Congress violates the principles of judicial review established in Marbury v. Madison (1803). B Congress may authorize a state to discriminate against interstate commerce without violating the Dormant Commerce Clause Doctrine. C The statute represents an impermissible delegation of Congress's Commerce Power. D The Dormant Commerce Clause Doctrine applies to both state and federal laws.

B Congress may authorize a state to discriminate against interstate commerce without violating the Dormant Commerce Clause Doctrine. Correct. In legislation enacted under its power to regulate interstate commerce, Congress may consent to state laws that would otherwise violate the Dormant Commerce Clause Doctrine. Prudential Life Insurance Co. v. Benjamin (1946).

Congress is considering a controversial bill that would impose new restrictions on private health care. Smith justifiably fears that, under the proposed new legislation, he will no longer be able to choose his own ophthalmologist. While politicians are hotly debating the bill, Smith has decided to file a lawsuit to preserve his constitutional rights. On what grounds should the court dismiss the lawsuit? A The court should dismiss any lawsuit filed by Smith because Congress's decision whether to pass the controversial proposed legislation is a political question. B Even if Smith is correct in claiming that the bill would violate his constitutional rights, a federal court cannot grant him relief because he has not yet suffered an invasion of a legally protected interest. C The court should dismiss the lawsuit because Congress has plenary power to regulate commerce, including health care. D Until Congress enacts the law, the legal issues involved are moot.

B Even if Smith is correct in claiming that the bill would violate his constitutional rights, a federal court cannot grant him relief because he has not yet suffered an invasion of a legally protected interest. Correct. The court cannot grant Smith relief because the matter is not yet "ripe" for decision. Until the legislation is passed and will apply to Smith, he has not yet suffered an injury. In addition, Smith lacks standing. One element of standing is that the plaintiff must have suffered an "injury in fact," which is an invasion of a legally protected interest. This injury must be both "concrete and particularized" and "actual or imminent." Lujan v. Defenders of Wildlife (1992).

In Gibbons v. Ogden (1824), Chief Justice Marshall wrote: "In our complex system, presenting the rare and difficult scheme of one general government, whose action extends over the whole, but which possesses only certain enumerated powers; and of numerous State governments, which retain and exercise all powers not delegated to the Union, contests respecting power must arise." What subject area within Constitutional Law most generally addresses the "contests" to which Chief Justice Marshall refers? A The Separation of Powers. B Federalism. C The State Action Doctrine. D The Requirement of a Republican Form of Government.

B Federalism. Correct. The subject of Federalism concerns the division of powers between the states and the federal government.

Carla Patterson called the police because she suspected that an intruder was in her house. The dispatcher told Patterson that a police officer would be there within 5 minutes. The police took 40 minutes to reach her house. By the time they arrived, Patterson had been beaten and robbed by the intruder. Patterson filed suit against the police department, alleging these facts and asserting that the police's failure to respond promptly to her call violated the Due Process Clause of the Fourteenth Amendment. The police department has filed a motion to dismiss.How should the court rule on the police department's motion? A Grant the motion, because 40 minutes is not an unreasonable response time for a busy police department. B Grant the motion, because the police department did not deprive Patterson of liberty or property within the meaning of the Due Process Clause. C Deny the motion, because the police department had an affirmative obligation to protect Patterson from harm. D Deny the motion, because the dispatcher's assurances that the police would arrive within 5 minutes created a legitimate expectation that the police would protect Patterson, and their failure to do so thus deprived Patterson of a property interest.

B Grant the motion, because the police department did not deprive Patterson of liberty or property within the meaning of the Due Process Clause. Correct. As noted above, the police department did not have an affirmative obligation to protect Patterson from private violence, and accordingly its failure to respond did not deprive Patterson of any interest protected by the Due Process Clause.

Many landlords include clauses in their leases precluding their tenants from displaying flags and banners. Some veterans' groups think all Americans should have the right to display the U.S. flag in their homes. They asked Congress to pass legislation prohibiting any clauses in leases that would restrict displaying the U.S. flag. Congress passed such a law after hearing substantial debate over whether the law was constitutional. The majority of Congress evidently concluded that the law did not violate the Constitution. Which of the following statements is correct? A Congress's determination that the law is constitutional is entitled to substantial deference by the courts because all members of Congress have sworn to uphold the Constitution. B If a court determines that the law comports with the Constitution, it must uphold the law even if it disagrees with the constitutional theories that were cited in Congress in support of the law. C The Supremacy Clause expressly requires the courts to invalidate the law if it is contrary to the Constitution. D Congress cannot regulate leases because leases are governed by state contract law.

B If a court determines that the law comports with the Constitution, it must uphold the law even if it disagrees with the constitutional theories that were cited in Congress in support of the law.

A New Jersey statute requires a license for the practice of psychology, which it defines as the "provision of psychological counseling for a fee." The statute provides that to qualify for a license, an applicant must possess a doctorate in psychology or a related field; must have at least two years of supervised professional experience under the direction of a licensed psychologist; and must pass an examination administered by the state Board of Psychology. The practice of psychology without a license is punishable as a misdemeanor. Priscilla Jones, who has a master's degree in psychoanalysis, and Robert Morgan, who has a master's degree in divinity, wish to counsel patients in much the same way that psychologists do, but they do not qualify for licenses to practice psychology under the New Jersey statute. After they were denied licenses, Jones and Morgan filed suit seeking a declaratory judgment that the licensing law violates the Due Process Clause of the Fourteenth Amendment.Should the district court invalidate the New Jersey licensing law? A No, because the Due Process Clause does not protect substantive rights, and the plaintiffs have not contended that they were entitled to a hearing before the state denied them a license. B No, because the Due Process Clause does not protect a fundamental right of freedom of contract, and the New Jersey law is rationally related to the state's interest in protecting the public health. C Yes, because the law interferes with the plaintiffs' right to practice their professions, and the law is not narrowly tailored to advance any compelling state interest. D Yes, because the law interferes with the plaintiffs' ability to enter into contractual arrangements with patients for counseling, and the state can advance its interest in protecting the public health by permitting patients to sue for fraud if counselors turn out to be unqualified.

B No, because the Due Process Clause does not protect a fundamental right of freedom of contract, and the New Jersey law is rationally related to the state's interest in protecting the public health. Correct. The Court has held that there is no fundamental right to freedom of contract, and that claims of interference with the freedom of contract should be reviewed for mere rationality. Because the state has a legitimate interest in protecting patients from counselors who do not have proper training, and because the licensing laws are rationally related to that interest, the law does not violate the Due Process Clause.

The Nevada legislature enacted a law requiring "all children between the age of 5 and 16 to attend school or its equivalent." The statute further provides that home schooling will be deemed the "equivalent" of school if the parent or person providing instruction has satisfactorily completed an on-line, 30-hour training course offered by the state. Margaret Kelly, a mother of an eight-year-old child who wishes to home school her child but has not completed the on-line training course, has filed a suit seeking a declaration that the Nevada law is unconstitutional.Should the court invalidate the statute as applied to Kelly? A No, because the statute is rationally related to the state's legitimate interest in increasing the number of qualified teachers. B No, because the state has a compelling interest in ensuring that children are well educated, and the statute, which ultimately permits parents to make decisions concerning the care, custody, control, and education of their children, is narrowly tailored to advance that interest. C Yes, because the state may not interfere with the liberty of parents to direct the upbringing and education of their children. D Yes, because the statute is not rationally related to any legitimate state interest.

B No, because the state has a compelling interest in ensuring that children are well educated, and the statute, which ultimately permits parents to make decisions concerning the care, custody, control, and education of their children, is narrowly tailored to advance that interest. Correct. Strict scrutiny is the correct level of scrutiny for this statute, which interferes with the plaintiff's fundamental right to direct the upbringing and education of her child; and because the statute arguably satisfies that standard, because it imposes only a modest burden on the plaintiff's right, and thus is narrowly tailored to advance the state's interest in ensuring that children are well educated.

Clinton v. New York (1998) concerned a statute that attempted to give the President a line-item veto over spending bills. Which of the following statements is false? A Proponents of the line-item veto argued that the line-item veto might help the President respond to excessive spending caused by congressional "logrolling." B Opponents of the line item veto argued that giving the President a line-item veto would violate Article II of the Constitution. C The Supreme Court held that the line-item veto was unconstitutional because it violated the requirements of Bicameralism and Presentment. D Many state governors have line-item veto powers.

B Opponents of the line item veto argued that giving the President a line-item veto would violate Article II of the Constitution. Correct. This statement is false. Opponents of the statute argued that the law violated Article I, not Article II. They asserted that it allowed the President to repeal statutes, something that ordinarily would require congressional involvement under Article I, section 7. They did not argue that the line-item veto would diminish the President's powers under Article II.

The Supreme Court held in Gibbons v. Ogden (1824) that Congress's power to regulate commerce is "plenary." According to modern precedents, what can Congress not do with this plenary power? A Prohibit individuals from engaging in particular commercial transactions. B Require otherwise inactive individuals to engage in particular commercial transactions. C Regulate how individuals engage in particular commercial transactions. D Subsidize individuals as they engage in particular commercial transactions.

B Require otherwise inactive individuals to engage in particular commercial transactions. Correct. The Supreme Court held in National Federation of Independent Business v. Sebelius (2012) that Congress could not rely on the Commerce Power to require an individual who was not otherwise engaged in commerce to purchase insurance using the Commerce Power because requiring commerce is not a form of "regulating" commerce.

The President has decided to fire the head of the Secret Service, an executive branch agency tasked with protecting the President and the President's family, following a series of mistakes and scandals by Secret Service officers. The head of the Secret Service seeks legal advice. Which of the following statements is true? A A statute may prohibit removal of the head of an executive agency prior to the expiration of a stated term of office. B The Constitution does not expressly give the President the power to remove senior executive branch officials, but the Supreme Court has held that this power is implied by the express power to appoint Officers of the United States. C The President can discharge executive branch officials only for good cause, including neglect, malfeasance, and incapacitation. D If the Senate confirmed the appointment of the head of the Secret Service, then the Senate also must concur in his or her removal.

B The Constitution does not expressly give the President the power to remove senior executive branch officials, but the Supreme Court has held that this power is implied by the express power to appoint Officers of the United States. Correct. The President has authority to remove the heads of executive agencies because the power to remove these officials is incident to the power to appoint their replacements. Myers v. United States (1926).

During a period of political tensions, the President of Russia decided to strike a symbolic blow at the United States by closing all of the American-owned McDonald's restaurants located in Moscow. Subsequently, when relations thawed, the President of the United States and the President of Russia agreed that the Russian government would pay the American restaurant owners a lump sum of $10,000 each in settlement of their losses caused by the closures. Many of the restaurant owners believe that this sum is insufficient to compensate them for their losses. Which of the following statements is true? A The United States government is liable to the restaurant owners for any losses exceeding $10,000. B The President of the United States may enter into international agreements settling claims by United States citizens against a foreign government even in the absence of express statutory or constitutional authority. C The actions of the President of the United States are lawful only if expressly authorized by federal statute or by treaty. D If the restaurant owners refuse to accept the $10,000, then they are not bound by the agreement between the President of the United States and the President of Russia.

B The President of the United States may enter into international agreements settling claims by United States citizens against a foreign government even in the absence of express statutory or constitutional authority. Correct. This is an accurate statement of the holding in Dames & Moore v. Regan (1981).

To avoid congestion on New Jersey thoroughfares, many truck drivers on both interstate and intrastate routes began using local roads for portions of their journeys across the state. In response, New Jersey enacted a statute preventing certain classes of large trucks from using streets in designated residential areas. The statute contains an exemption for New Jersey state and local government vehicles. Interstate trucking firms have challenged the constitutionality of the law. Which of the following statements is correct? A The exemption violates the Dormant Commerce Clause Doctrine because it discriminates against interstate commerce. B The exemption does not violate the Dormant Commerce Clause doctrine so long as the benefits of keeping the trucks out of residential neighborhoods outweigh the inconveniences to the flow of interstate commerce. C New Jersey is permitted to restrict large trucks from streets in residential areas because Congress has no power to regulate local streets except through conditional spending. D New Jersey is permitted to grant an exemption to state vehicles because Congress has no power to regulate state and local government vehicles.

B The exemption does not violate the Dormant Commerce Clause doctrine so long as the benefits of keeping the trucks out of residential neighborhoods outweigh the inconveniences to the flow of interstate commerce. Correct. If the benefits of keeping the trucks out of residential neighborhoods outweigh the inconveniences to the flow of interstate commerce, the law does not violate the Dormant Commerce Clause Doctrine's prohibition of state laws that impose a burden on interstate commerce that is excessive in relation to legitimate local interests. Southern Pacific v. Arizona (1945); Kassel v. Consolidated Freightways Corp. (1981). Discrimination against interstate commerce in favor of the local government does not violate the Dormant Commerce Clause Doctrine. United Haulers Ass'n, Inc. v. Oneida-Herkimer Solid Waste Management Authority (2007).

When Congress relies on its Commerce Power to pass a statute, it often expressly makes some connection to commerce an element of the statute. In Lopez v. United States (1995), however, the Supreme Court recognized that such an element is not necessary for a statute to be constitutional. Which of the following cases best supports this proposition? A NLRB v. Jones & Laughlin Steel Corp. (1937) B Wickard v. Filburn (1942) C United States v. Darby (1941) D Katzenbach v. McClung (1964)

B Wickard v. Filburn (1942) Correct. A jurisdictional element is a term in a federal statute that expressly links the regulated conduct to interstate commerce. The statute at issue in Wickard v. Filburn (1942) regulated farm production without expressly linking this production to interstate commerce.

The South Dakota legislature enacted a law that makes it a crime for a doctor to perform an abortion after the 26th week of pregnancy, except when necessary to protect the life of the mother. A woman in her 27th week of pregnancy who wishes to obtain an abortion to avoid the risk of paralysis has filed suit challenging the constitutionality of the law. At trial, the state introduces undisputed evidence that shows that more than 90 percent of babies born after 26 weeks of gestation survive.Should the district court invalidate the law? A Yes, because the state cannot ban abortion before the final trimester of the typical pregnancy, which begins two-thirds of the way through a 40-week gestation, or in the 27th week of pregnancy. B Yes, because the law does not contain an exception for cases in which abortion is necessary to protect the health of the mother. C No, because the state can ban abortion after the point of viability as long as it creates an exception for cases in which abortion is necessary to protect the life of the mother. D No, because the law does not impose an undue burden on the right to obtain an abortion.

B Yes, because the law does not contain an exception for cases in which abortion is necessary to protect the health of the mother. Correct. Even assuming that 26 weeks is the point of viability, the state must preserve the right of a woman to obtain an abortion when "it is necessary, in appropriate medical judgment, for the preservation of the life or health of the mother." Casey.

Congress has enacted a comprehensive scheme of laws governing radio communications over certain frequencies. Smith is an organic farmer in Oregon who lives on his own farm and grows Belgian endives. In violation of one provision of the laws, Smith uses a low-powered radio transmitter set to a prohibited frequency to chat with his parents, who live on the same farm but in a separate house. It is undisputed that Smith's radio signals are too weak to reach beyond the boundaries of his farm or interfere with any other radio signal. Smith contends that the provision at issue is unconstitutional because it is beyond the reach of Congress's Commerce Power. Is the provision within Congress's commerce power? A No. Congress cannot regulate Smith's radio signal if his signal, even when aggregated with all other similar low-powered radio signals throughout the United States, would be too weak to have a substantial effect on interstate commerce. B Yes. Congress had power to enact the provision at issue if the provision is necessary to ensure that a comprehensive scheme for regulating interstate commerce is not undercut, even if the provision by itself would reach conduct that does not have a substantial effect on interstate commerce. C No. Congress cannot regulate Smith's radio signals because they are not used for a commercial purpose. D Yes. Congress could rationally conclude that Smith's radio signal by itself has an effect on interstate commerce even if Congress knew that the signals were too weak to reach beyond the boundaries of his farm and could not identify any way in which the signal conceivably might affect interstate commerce.

B Yes. Congress had power to enact the provision at issue if the provision is necessary to ensure that a comprehensive scheme for regulating interstate commerce is not undercut, even if the provision by itself would reach conduct that does not have a substantial effect on interstate commerce. Correct. Because Congress has created a comprehensive scheme that directly regulates economic, commercial activity, a court cannot excise individual applications of the scheme even when they apply to local, non-economic conduct. Gonzales v. Raich (2005).

Which of the following assertions accurately state the current understanding of the Due Process Clause of the Fourteenth Amendment? 1. To survive scrutiny under the Due Process Clause, all state laws that limit our freedom of action in any way must be narrowly tailored to advance a compelling government interest. 2. To survive scrutiny under the Due Process Clause, state laws that directly and substantially interfere with fundamental rights usually must be narrowly tailored to advance a compelling government interest. 3. To survive scrutiny under the Due Process Clause, state laws that interfere with fundamental rights will be upheld unless they impose an undue burden on a fundamental right. 4. To survive scrutiny under the Due Process Clause, state laws that limit our freedom of action but that do not interfere with fundamental rights must be rationally related to a legitimate government interest. 5. State laws that limit our freedom of action but that do not interfere with fundamental rights are not subject to scrutiny under the Due Process Clause of the Fourteenth Amendment. A 1 and 2 B 3 and 4 C 2 and 4 D 2 and 5

C 2 and 4 Correct. For the reasons noted above and below, assertions 2 and 4 are correct, and the other assertions are incorrect.

Which of the following statements about the Court's decision in Bush v. Gore is false: A A majority of the Justices concluded that the recount procedures ordered by the Florida Supreme Court violated the Equal Protection Clause. B A majority of the Justices concluded that a revised recount plan that would comply with the Equal Protection Clause could not be carried out before the date identified in a federal statute for the selection of electors for President. C A majority of the Justices concluded that voting procedures that entail the punching of a card with a stylus violate the Equal Protection Clause. D A majority of the Justices concluded that the Constitution does not guarantee a right to vote for electors for President unless the state legislature selects a statewide election as the means for choosing members of the electoral college.

C A majority of the Justices concluded that voting procedures that entail the punching of a card with a stylus violate the Equal Protection Clause. Correct. The statement false. Although the particular voting method used in many counties in the state contributed to the problem of determining voter intent, the Court did not conclude that such voting procedures violated the Equal Protection Clause. Instead, the Court suggested that the recount procedures did not specify uniform statewide rules for counting votes, which could have lead to unequal treatment. In addition, the Court expressly stated that its "consideration [was] limited to the present circumstances."

Congress recently enacted a far-reaching law aimed at establishing new educational standards for public and private high schools. While the bill was pending, a debate arose about whether Congress had power under the Commerce Clause to enact the legislation. Members of Congress divided on this question along party lines, with Democrats arguing that the law was constitutional and Republicans arguing the opposite. Which of the following potential plaintiffs is MOST LIKELY to have standing to challenge the constitutionality of the law in federal court? A A Republican member of Congress who voted against the law. B A college admissions officer who believes that the new standards will not prepare most high school graduates properly for college, making successful recruiting more difficult. C A middle school student who will enter high school next semester and will be subject to the new standards. D A grandmother who gives her daughter and son-in-law money to help them pay her grandson's tuition at a private high school.

C A middle school student who will enter high school next semester and will be subject to the new standards. Correct. Although the middle school student has not yet suffered an injury because of the standards (given that they only apply to high schools), an injury is "imminent" because the student will enter high school next semester. Lujan v. Defenders of Wildlife (1992).

Which of the following hypothetical laws is most likely to violate the Dormant Commerce Clause Doctrine's Uniform National Standard Test? A A state law specifying that motor vehicles cannot turn right at a red traffic light, contrary to the laws of all other states. B A federal law establishing agricultural standards for Hawaii that are different from the agricultural standards for other states. C A state law mandating the wearing of state-approved bicycle helmets for the entire distance of all organized bicycle tours over 100 miles in length. D A state law requiring anyone selling milk in the state to pay the cost of safety inspections.

C A state law mandating the wearing of state-approved bicycle helmets for the entire distance of all organized bicycle tours over 100 miles in length. Correct. A uniform national standard may be necessary if allowing multiple states to regulate a subject could result in conflicting regulations with which compliance would be impossible. Wabash, St. L. & P. Ry. Co. v. Illinois (1886). Bicycle tours of over 100 miles in length may extend into more than one state. Accordingly, if a state tries to regulate the entire distance of a tour, its laws may conflict with those of another state into which the tour extends, making compliance impossible. For example, another state may only approve a different type of helmet.

What is the theory of the "extra-constitutional origin of the foreign affairs power" as addressed in United States v. Curtiss-Wright Export (1936)? A A theory that the Necessary and Proper clause in Article I and various provisions in Article II give Congress and the President implied powers over foreign affairs. B A theory that the Bill of Rights does not apply outside of the territory of the United States except with respect to U.S. citizens. C A theory that the national executive power with respect to foreign affairs was not delegated to the federal government by the States but was instead transferred from the United Kingdom to the United States when the United States became an independent nation. D A theory that the Constitution, federal laws, and federal treaties are subordinate to generally accepted principles of international law.

C A theory that the national executive power with respect to foreign affairs was not delegated to the federal government by the States but was instead transferred from the United Kingdom to the United States when the United States became an independent nation. Correct. This statement reflects the holding in United States v. Curtiss-Wright Export (1936). Under this theory, the United States may exercise not only the powers that the Constitution expressly grants, but also other foreign affairs powers enjoyed by all sovereigns.

After several accidents in which small, remote-controlled drone aircraft crashed into electric power lines, the New Jersey state legislature passed a law requiring completion of a twelve-hour training course before flying any drone (including drones marketed as toys) within the state. The police recently ticketed several out-of-state tourists who were using toy drones while visiting one of the state's famous beaches because the tourists had not completed the training course in violation of the state law. The tourists, who had never heard of the New Jersey law, think that the tickets are unfair and have decided to challenge them. Their conduct did not violate current federal law, which restricts only commercial drone use. Which of the following statements is correct? A The New Jersey law violates the Dormant Commerce Clause Doctrine. Drones must be regulated exclusively at the national level because they are readily transported from state to state. B The New Jersey law violates the Dormant Commerce Clause Doctrine because it has a disparate impact on out-of-state residents who are less likely to have heard about the law than in-state residents. C Congress could enact a new federal law on the subject of drones that would preempt the New Jersey law even if it would be possible for drone users to comply with both the new federal law and the state law. D States cannot regulate drones because, in the aggregate, drones substantially affect interstate commerce.

C Congress could enact a new federal law on the subject of drones that would preempt the New Jersey law even if it would be possible for drone users to comply with both the new federal law and the state law. Correct. A federal law may preempt a state law, even in the absence of any kind of conflict, if Congress intended the federal law to occupy the entire field of the law. Silkwood v. Kerr-McKee (1984).

Congress has enacted a law giving broad discretion to the Department of Energy to issue regulations concerning solar power. Some members of Congress now disagree with how the Department is exercising this discretion. Which of the following responses by Congress would be unconstitutional? A Threatening to reduce the Department of Energy's budget in the future unless the Department changes its regulations. B Requiring Department of Energy officials to testify about the regulations that the Department has issued and confronting these officials about their policy decisions. C Enacting a new law that would allow Congress, by concurrent resolution, to nullify certain Department of Energy regulations with which Congress disagrees. D Passing a bill that would amend the law to reduce the Department of Energy's discretion when issuing regulations in the future.

C Enacting a new law that would allow Congress, by concurrent resolution, to nullify certain Department of Energy regulations with which Congress disagrees. Correct. This response would be unconstitutional. Congress may exercise legislative power only by acting pursuant to the Bicameralism and Presentment requirements in Article I, section 7. A concurrent resolution is a resolution approved by both houses of Congress but not presented to the President for approval. Nullifying Department of Energy regulations would be a form of legislative action because it would alter legal rights and because it is an action that generally would be accomplished by passing laws. INS v. Chadha (1983).

Washington. D.C. attorney Smith had been a thorn in the President's side for several years, constantly filing lawsuits challenging executive actions and criticizing the President on television talk shows. Recently, the IRS has been investigating a number of Smith's financial transactions. Word of these investigations has leaked to the press and have greatly damaged Smith's reputation. Smith believes, with probable cause, that the President ordered the IRS to investigate him as retaliation for Smith's irritating actions. Which of the following statements is true? A The President can avoid any civil or criminal liability potentially arising out of the investigation of Smith by asserting Executive Privilege. B The President is not immune from civil liability to Smith if the President has acted improperly, but Smith must wait until the President leaves office before pursuing a civil lawsuit against the President. C Even if what Smith believes is true, Smith cannot recover damages from the President. D The president has a right to delay any and all litigation pursued against him while serving as President.

C Even if what Smith believes is true, Smith cannot recover damages from the President. Correct. The President has absolute immunity from civil liability for official acts taken while President. Nixon v. Fitzgerald (1982).

In 1821, Congress passed an act to incorporate the Columbian College in the District of Columbia (later renamed the George Washington University). The Act did not specify why Congress believed that it had power to grant a corporate charter to the University. Which of the following statements is false? A The Constitution contains no provision expressly mentioning the power to grant corporate charters, but this power may be implied as necessary and proper for carrying out Congress's enumerated powers. B Congress has exclusive legislative authority over the District of Columbia. C The Act was unconstitutional when it was adopted. D Congress could have subsidized the Columbian College using its Spending Power.

C The Act was unconstitutional when it was adopted. Correct. This statement is false. The law was not unconstitutional when adopted. Congress had power to enact the law under Article I, section 8, clause 17, and the Necessary and Proper Clause. McCulloch v. Maryland (1819).

In Gibbons v. Ogden (1824), what was Justice Johnson's point when he said, "I cannot overcome the conviction, that if the licensing act was repealed tomorrow, the rights of the appellant to a reversal of the decision complained of would be as strong as it is under this license"? A Chief Justice Marshall's opinion for the Court was incorrect in holding that the federal licensing law preempted the state licensing law. B The Dormant Commerce Clause Doctrine lacks a textual basis and is unnecessary. C The Commerce Clause would render the state law invalid even if Congress had not preempted it by federal legislation. D The states and Congress have concurrent power to regulate commerce.

C The Commerce Clause would render the state law invalid even if Congress had not preempted it by federal legislation. Correct. Justice Johnson believed that when the Commerce Clause gave Congress the power to regulate commerce it deprived the states of the power to enforce the kind of law New York had passed in Gibbons. In essence, this is what the Dormant Commerce Clause Doctrine ultimately came to say.

The Court in Griswold v. Connecticut reversed the appellants' convictions because: A They were convicted for engaging in private, intimate activity that the government lacks the power to regulate. B The Connecticut statute violated not the Fourteenth Amendment, as the appellants contended, but the First, Third, Fourth, and Fifth Amendments, which apply of their own force to actions by the states. C The Connecticut statute was inconsistent with a right of privacy protected by the implications of several provisions in the Bill of Rights, which apply to the states through the Due Process Clause of the Fourteenth Amendment. D The Connecticut statute was an "uncommonly silly law."

C The Connecticut statute was inconsistent with a right of privacy protected by the implications of several provisions in the Bill of Rights, which apply to the states through the Due Process Clause of the Fourteenth Amendment. Correct. The Court concluded that the "penumbras" of various provisions in the First, Third, Fourth, and Fifth Amendments together create a right of privacy, and that the right of privacy applies to state action through the force of the Due Process Clause of the Fourteenth Amendment.

In Nixon v. United States (1993), former federal judge Walter Nixon challenged his impeachment, conviction, and removal from office. His argument focused on Article I, section 3, clause 4, which provides: "The Senate shall have the sole Power to try all Impeachments." Judge Nixon contended that the Senate's use of a committee to conduct part of the trial (rather than the whole Senate) was inconsistent with this clause. Which of the following statements best characterizes the Supreme Court's decision in the case? A The Senate's use of a committee in trying Judge Nixon did not violate Article I, section 3, clause 4. B Whether the Senate's use of a committee in trying Judge Nixon violated Article I, section 3, clause 4, is not a constitutional question. C The Court cannot determine whether the Senate's use of a committee in trying Judge Nixon violated Article I, section 3, clause 4. D The Court effectively overruled its decision in Baker v. Carr (1962).

C The Court cannot determine whether the Senate's use of a committee in trying Judge Nixon violated Article I, section 3, clause 4. Correct. This is an accurate statement of what the Court decided. A court cannot decide political questions.

The Court in Lawrence v. Texas invalidated the Texas law because: A The Due Process Clause protects a fundamental right to engage in gay sex, and although the statute furthered a compelling state interest in protecting morality, it was not narrowly tailored to advance that interest. B The state never has a legitimate interest in protecting morality, which was the only conceivable justification for the statute. C The Due Process Clause protects the liberty of gay people to engage in consensual sex in the privacy of the home, and the Texas statute did not further any legitimate state interest that could justify interference with that right. D The law impermissibly treated gay people and heterosexual people differently, in violation of the Equal Protection Clause.

C The Due Process Clause protects the liberty of gay people to engage in consensual sex in the privacy of the home, and the Texas statute did not further any legitimate state interest that could justify interference with that right. Correct. The Court apparently concluded that the "petitioners were free as adults to engage in the private conduct in the exercise of their liberty under the Due Process Clause"—though it did not make clear whether such liberty is a fundamental right—and concluded that the law does not further any legitimate state interest to justify the interference with that liberty.

Congress recently has become concerned about whether major cities have sufficient quantities of key medicines to treat their residents in the event of a major epidemic. Most of the medicines at issue are produced by a few domestic manufacturers and then shipped throughout the nation. Congress regulates the shipments but does not require pharmacies to maintain any particular level of inventory. Congress is now thinking about passing a law requiring pharmacies in a number of cities to make quarterly reports about the stock of key medicines that they have on hand. A proponent of the legislation asks you for assistance in arguing that Congress has the power to pass such a law. Which clause of the Constitution is least relevant to the debate? A The Necessary and Proper Clause. B The Commerce Clause. C The General Welfare Clause. D The Tenth Amendment.

C The General Welfare Clause. Correct. The General Welfare Clause is not relevant to the debate about whether Congress has the power to pass this law. The General Welfare Clause implicitly grants Congress power to spend money to promote the general welfare, so long as the spending does not violate any other constitutional limitation. Congress, therefore, may appropriate funds for purposes unrelated to the specific powers enumerated in Article I, section 8. United States v. Butler (1936). This debate, however, does not concern the spending of money.

An Illinois law allows its citizens to vote in Illinois elections, but does not allow citizens from other states to vote in Illinois elections. An Indiana resident who wishes to vote in the next Illinois election (without becoming an Illinois resident) has challenged the law on grounds that it violates the Privileges and Immunities Clause. Which of the following statements is correct? A The Indiana resident has not suffered an injury in fact and therefore lacks standing to challenge the Illinois law. B A state may discriminate against citizens of other states with respect to non-fundamental privileges such as hunting and fishing licenses, but not with respect to fundamental rights such as voting. C The Illinois law discriminates against citizens of other states but does not violate the Privileges and Immunities Clause. D The Constitution contains a clause that expressly authorizes states to limit suffrage in local elections to their own citizens.

C The Illinois law discriminates against citizens of other states but does not violate the Privileges and Immunities Clause. Correct. The Supreme Court said in Baldwin v. Fish & Game Commission (1978) that the Privilege and Immunities Clause does not require a state to open its polls to persons who vote in other states. Although this law clearly discriminates against citizens of other states, the Privileges and Immunities Clause only prohibits discrimination against out-of-state residents or citizens (but not corporations or aliens) if the discrimination might jeopardize interstate harmony, unless the discrimination is necessary to promote a substantial state interest. Denying out-of-state citizens the right to vote in local elections is not likely to jeopardize interstate harmony because it is a well-accepted practice designed to further a state's substantial interest in having local governance.

The Court in Slaughter-House Cases held that the Louisiana law did not violate the Privileges or Immunities Clause of the Fourteenth Amendment because: A The Privileges or Immunities Clause of the Fourteenth Amendment does not protect rights that are not explicitly mentioned in the Constitution. B The Louisiana law applied only to citizens of Louisiana, and not to citizens of the United States. C The Privileges or Immunities Clause of the Fourteenth Amendment protects only those rights that owe their existence to the federal government, its national character, its Constitution, or its laws, a category that does not include the right to practice one's profession without interference. D The Privileges or Immunities Clause of the Fourteenth Amendment requires only that when a state grants a right to its own citizens, it must also grant the same right to citizens of other states when they are within the state's jurisdiction.

C The Privileges or Immunities Clause of the Fourteenth Amendment protects only those rights that owe their existence to the federal government, its national character, its Constitution, or its laws, a category that does not include the right to practice one's profession without interference. Correct. The Court reasoned that only "the privileges and immunities of the citizens of the United States" are "placed by this Clause under the protection of the Federal Constitution," and that those privileges and immunities are those that owe "their existence to the Federal government, its National character, its Constitution, or its laws."

You are an intern for a U.S. senator from West Virginia. Some of the senator's constituents have asked for federal governmental assistance in recovering from localized flooding of the Ohio River. The senator wants to propose an amendment to a pending spending bill that would provide federal funds to the West Virginia citizens and businesses whose property was damaged in the flood. A staffer in the senator's office asks you for advice about whether Congress can constitutionally spend money for this purpose. What advice do you provide? A Congress can provide financing only for subjects linked, directly or indirectly, to interstate commerce. B Congress has plenary power to spend money that is unreviewable by the courts. C The choice of how to spend money belongs to Congress, unless the choice is clearly wrong, a display of arbitrary power. D Adding the spending provision to the bill is not unconstitutional but the President might use a line-item veto to cancel the spending.

C The choice of how to spend money belongs to Congress, unless the choice is clearly wrong, a display of arbitrary power. Correct. The Court announced this standard in Helvering v. Davis (1937), the case upholding the old-age benefits provisions of the Social Security Act, which imposed a tax on employees to fund payments to older citizens. This exact language has been repeated in subsequent cases, see, e.g., Bowen v. Gilliard, 483 U.S. 587, 598 (1987), and was not contested in National Federation of Independent Business v. Sebelius (2012), which relied on Helvering.

Fred Johnson failed to pay a $50 parking ticket that he received for parking in a no-parking zone in Phoenix, Arizona. Because of his failure to pay, the city assessed a $500 fine. Johnson has challenged the imposition of the fine, asserting that it violates the Eighth Amendment, which provides in relevant part that "Excessive bail shall not be required, nor excessive fines imposed ***." The city has defended on the ground that the Excessive Fines Clause of the Eighth Amendment does not apply to the states.How should the court rule on the question whether the Clause applies to the states? A The court should conclude that the Excessive Fines Clause does not apply to the states because it is not a right that owes its existence to the federal government, its national character, its Constitution, or its laws. B The court should conclude that the Excessive Fines Clause applies to the states because the Due Process Clause of the Fourteenth Amendment incorporates every provision in the first eight amendments to the Constitution—nothing more and nothing less. C The court should conclude that the Due Process Clause of the Fourteenth Amendment incorporates the Excessive Fines Clause because a prohibition on excessive fines, and other forms of extreme punishment, are fundamental to our scheme of ordered liberty and deeply rooted in our Nation's history and tradition. D The court should conclude that the Due Process Clause of the Fourteenth Amendment does not protect any substantive rights, but that the Privileges or Immunities Clause of the Fourteenth Amendment protects all constitutionally enumerated rights, including the right to be free from excessive fines, against state infringement.

C The court should conclude that the Due Process Clause of the Fourteenth Amendment incorporates the Excessive Fines Clause because a prohibition on excessive fines, and other forms of extreme punishment, are fundamental to our scheme of ordered liberty and deeply rooted in our Nation's history and tradition. Correct. It properly expresses the modern standard for determining whether a provision of the Bill of Rights is incorporated by the Due Process Clause and thus applies to state action. The Court confirmed this in its decision in Timbs v. Indiana (2018).

Roger Jones and Patrick McKinley were arrested for having consensual sex in a public restroom at Penn Station in New York City. They were indicted under a New York statute that makes it a misdemeanor to engage in sexual relations "in any place of public accommodation or any place open to the public." They filed a motion to dismiss the indictment on the ground that the statute violates the Due Process Clause of the Fourteenth Amendment.How should the court rule? A The court should grant the motion because the state cannot make sexual conduct a crime. B The court should grant the motion because the law does not further any legitimate state interest. C The court should deny the motion because the Due Process Clause does not protect a fundamental right to engage in sexual conduct in public places, and the statute is rationally related to the legitimate government interest in ensuring public order. D The court should deny the motion because the Constitution does not protect a right of homosexuals to engage in sodomy.

C The court should deny the motion because the Due Process Clause does not protect a fundamental right to engage in sexual conduct in public places, and the statute is rationally related to the legitimate government interest in ensuring public order. Correct. The Court in Lawrence focused on the fact that the conduct there at issue took place in private at home, and emphasized that sexual conduct in public would raise a different question. Because there presumably is no fundamental right to engage in sex in public, the appropriate level of scrutiny is rational-basis review. The statute survives that level of scrutiny because it is rationally related to the legitimate interest in ensuring public order (among other interests).

Arkansas law does not impose any requirements for the construction or design of caskets for burial, but it provides that sales of caskets to the public in Arkansas may be made only by a state-licensed funeral director and only at a state-licensed funeral home. Monks at a Catholic monastery build wooden caskets and would like to sell them at prices substantially lower than those offered by state-licensed funeral homes. After the state Board of Funeral Services ordered the monastery to stop selling caskets in Arkansas, the monastery filed suit seeking a declaration that the Arkansas law violates the Due Process Clause of the Fourteenth Amendment. State officials have defended on the ground that the law advances the state's interest in protecting the funeral industry from competition.How should the court rule? A The court should hold that the law does not violate the Due Process Clause because state regulation of economic matters cannot give rise to a claim under that Clause. B The court should hold that the law does not violate the Due Process Clause because the court must accept the state's contention that the law advances a legitimate state interest. C The court should hold that the law violates the Due Process Clause if it concludes that the mere desire to protect a particular industry from economic competition is not a legitimate state interest and that the law is not rationally related to any other legitimate government interest. D The court should hold that the law violates the Due Process Clause because the law unfairly excludes the monastery from participating freely in the market for casket sales.

C The court should hold that the law violates the Due Process Clause if it concludes that the mere desire to protect a particular industry from economic competition is not a legitimate state interest and that the law is not rationally related to any other legitimate government interest. Correct. The law, which does not interfere with any fundamental right, triggers rational-basis review, which asks whether the law is rationally related to a legitimate government interest. To survive rational-basis review, the law need only be related to some conceivable interest, even one that the state did not claim to advance. But if economic protectionism is not a legitimate government interest, then the law can be upheld only if it is rationally related to some other legitimate interest. And the law would fail rational-basis review, and thus violate the Due Process Clause, if it is not rationally related to any conceivable legitimate government interest.

Concerned about large budget deficits, Congress has enacted a statute creating an "executive committee" for the U.S. Department of Treasury. The committee consists of the Secretary of the Treasury, two members of the Senate (one from each party), and two members of the House (also one from each party). The statute provides that this committee, by majority vote, can block any non-defense-related spending and by unanimous vote can block any defense-related spending. The statute also allows Congress to replace the congressional members of the committee at any time. Which of the following statements is true? A Even if the statute is unconstitutional, no one other than the President and the Secretary of the Treasury would have standing to challenge its constitutionality in federal court. B The statute is constitutional, but only with respect to new spending authorized by Congress after enactment of the statute. C The law is unconstitutional because it violates the separation of powers. D The law is constitutional with respect to both new spending and spending authorized before its enactment.

C The law is unconstitutional because it violates the separation of powers. Correct. The statute is unconstitutional because it violates the separation of powers. The specific problem is that legislation cannot give executive powers to officials whom Congress can remove—such as the four committee members from the House and Senate—other than by impeachment and conviction. Bowsher v. Synar (1986). Sequestering authorized spending is an executive function; as described in Clinton v. City of New York (1998), the President historically has exercised this power.

Article I, section 8, clause 3, grants Congress power to regulate Commerce "among the several States." How does the quoted phrase restrict Congress's power? A The phrase restricts Congress to regulating only transactions involving buyers and sellers in different states. B The phrase restricts Congress to regulating only transactions in which goods are shipped or traded across state lines. C The phrase restricts Congress to regulating only that commerce which concerns more than one state. D The phrase prevents Congress from regulating any transaction in which the buyer and seller are located in the same state and the goods never leave the state.

C The phrase restricts Congress to regulating only that commerce which concerns more than one state. Correct. Chief Justice Marshall wrote in Gibbons v. Ogden (1824) that the phrase restricts Congress's regulatory power to commerce "which concerns more States than one." Commerce may "concern" more than one state even if it occurs solely within one state.

According to the Court in Gibbons v. Ogden (1824), what limits Congress's plenary power to regulate commerce among the states? A The Tenth Amendment. B The Necessary and Proper Clause. C The wisdom and the discretion of Congress. D An implied standard of reasonableness.

C The wisdom and the discretion of Congress. Correct. In his opinion for the Court, Chief Justice Marshall said: "The wisdom and the discretion of Congress, their identity with the people, and the influence which their constituents possess at elections, are, in this, as in many other instances, as that, for example, of declaring war, the sole restraints on which they have relied, to secure them from its abuse."

In Carter v. Carter Coal (1936), the Supreme Court held that provisions of the Bituminous Coal Conservation Act of 1935 regulating the wages and hours of each coal miner exceeded Congress's power under the Commerce Clause. The Court reasoned that an individual miner's production of coal does not have a direct effect on interstate commerce. Why would this reasoning be insufficient for concluding that Congress lacked power to enact the provision under more recent Commerce Clause precedents? A Under current law, the Court would consider whether Congress rationally could conclude that the aggregate production of all affected miners might have an economic effect on interstate commerce. B Under current law, the Court would consider whether Congress reasonably could conclude that the aggregate production of all affected miners might have a direct effect on interstate commerce. C Under current law, the Court would consider whether Congress rationally could conclude that the aggregate production of all affected miners might have a substantial effect on interstate commerce. D Under current law, the Court would consider whether Congress rationally could conclude that the average production of all affected miners might have an indirect effect on interstate commerce.

C Under current law, the Court would consider whether Congress rationally could conclude that the aggregate production of all affected miners might have a substantial effect on interstate commerce. Correct. This answer states the current doctrine. United States v. Lopez (1995). The Court overruled Carter v. Carter Coal (1937) in United States v. Darby (1942).

Which of the following issues is the Supreme Court LEAST LIKELY to characterize as a non-justiciable political question under the factors identified in Baker v. Carr (1962)? A Whether a lawyer nominated for a federal judgeship has the necessary training and experience to serve as a federal judge. B Whether the President alone can terminate a treaty with a foreign nation. C Whether Congress has exceeded the limitations of Article III in a statute defining the Supreme Court's original jurisdiction. D Whether Congress had a proper basis for declaring war following an armed attack by a foreign nation.

C Whether Congress has exceeded the limitations of Article III in a statute defining the Supreme Court's original jurisdiction.Correct. This is not a political question. Indeed, the Supreme Court decided such an issue in Marbury v. Madison (1803).

In deciding whether Congress has power under the Commerce Clause to regulate an economic activity occurring wholly within one state, which factors will the courts consider? A Whether the actual purpose of the legislation is to regulate interstate commerce. B The wisdom, workability, or fairness of the legislation. C Whether the activity in the aggregate substantially affects interstate commerce. D All of the above.

C Whether the activity in the aggregate substantially affects interstate commerce. Correct. When determining whether the local activity that Congress seeks to regulate has the requisite connection to interstate commerce, the Court considers all of the regulated activity in the aggregate. Wickard v. Filburn (1942).

The Supreme Court has held that federal courts lack authority to issue "advisory opinions" on the proper resolution of abstract legal questions divorced from an actual dispute between adverse parties. What is the constitutional basis for this conclusion? A The Fifth Amendment's Due Process clause requires courts to resolve all legal issues through an adversarial procedure. B Experience has shown that issues are decided better in the context of actual cases and controversies. C Under Article II, if an advisory opinion is needed, the President may require "the principal Officer in each of the executive Departments" to provide their opinions "upon any Subject relating to the Duties of their respective Offices." D Article III limits the subject matter jurisdiction of federal courts to various types of "cases" and "controversies."

D Article III limits the subject matter jurisdiction of federal courts to various types of "cases" and "controversies." Correct. The Supreme Court held in Muskrat v. United States (1911) that the limitation of federal court subject matter jurisdiction to "cases" and "controversies" in Article III, section 2, clause 1, prevents the federal courts from issuing advisory opinions.

Which of the following statements about the Bicameralism and Presentment requirements in Article I, section 7 is true? A Because of the Bicameralism and Presentment requirements, no bill can become a law unless the bill is passed by both houses of Congress and signed by the President. B Although congressional resolutions may set rules for the governance of Congress's internal operations, congressional resolutions cannot have the effect of law because of the Bicameralism and Presentment requirements. C The Bicameralism and Presentment requirements preclude Executive Branch agencies from promulgating rules and regulations that have the force of law. D Congress cannot evade the Bicameralism and Presentment requirements by allowing the President, or either house of Congress, acting unilaterally, to veto portions of laws once they have been presented to the President and approved by the President.

D Congress cannot evade the Bicameralism and Presentment requirements by allowing the President, or either house of Congress, acting unilaterally, to veto portions of laws once they have been presented to the President and approved by the President. Correct. This statement is true. The Supreme Court has invalidated attempts to give the House and Senate "legislative vetoes," see INS v. Chadha (1983), and to give the President a "line-item" veto, see Clinton v. New York (1998).

Within months after the Supreme Court's decision in Hamdan v. Rumsfeld (2006), Congress enacted the Military Commissions Act of 2006. This Act took away from enemy detainees in U.S. military custody certain rights that the Supreme Court in Hamdan had held were provided by the Geneva Conventions, which are international treaties to which the United States is a party. Which of the following statements is false? A In enacting this new legislation, Congress was not bound by the Geneva Conventions. B Other nations may claim that the United States has violated the Geneva Conventions and take countermeasures. C Although Congress has effectively countermanded the Supreme Court, its actions do not violate the principle of judicial review. D If a detainee's lawsuit reaches the Supreme Court, the principle of stare decisis would direct the Supreme Court to adhere to the outcome in Hamdan.

D If a detainee's lawsuit reaches the Supreme Court, the principle of stare decisis would direct the Supreme Court to adhere to the outcome in Hamdan. Correct. This statement is false. The Supreme Court has held that treaties and statutes stand on equal footing, and whichever comes second in time is controlling. See Whitney v. Robinson (1888). The Supreme Court would be bound by the new legislation rather than by its precedent interpreting the Geneva Conventions.

The Board of Regents of the University of Texas, a state-chartered and state-operated school, fired Professor Albert Peterson after he received poor teaching evaluations. He filed suit in federal district court against the University, asserting that he was entitled under the Due Process Clause of the Fourteenth Amendment to a hearing before being terminated to explain why he received poor evaluations and why he should not be fired. Professor Peterson had been employed at the University for the last five years under a series of one-year contracts. These contracts provided: "The Board of Regents shall endeavor to preserve the continuity of the faculty, but Professor Peterson serves at the will of the Board."Should the district court conclude that he was entitled to a hearing before being terminated? A Yes, because the termination deprived Professor Peterson of the liberty to practice his profession and a hearing would not have been unduly burdensome to the University. B Yes, because Professor Peterson had a property interest in his job and the Due Process Clause required some kind of hearing. C No, because although Professor Peterson had a property interest in his job, a hearing probably would not have affected the Board's decision to terminate Peterson. D No, because the termination did not deprive Professor Peterson of any liberty or property interest protected by the Due Process Clause.

D No, because the termination did not deprive Professor Peterson of any liberty or property interest protected by the Due Process Clause. Correct. The Due Process Clause does not protect a right to practice one's profession and Peterson's at-will status meant that he did not have a property interest in continued employment. Absent the deprivation of a protected liberty or property interest, the Due Process Clause does not require a pre-termination hearing.

The First Congress passed 96 important acts. Among other things, these acts:(1) imposed federal taxes on imported goods(2) shaped the executive branch by establishing the Departments of War and Treasury; and(3) created federal courts inferior to the Supreme Court.Which of these acts would have been valid under the Articles of Confederation? A Only (1) B Only (2) C Only (2) and (3) D None

D None Correct. Under the Articles of Confederation, Congress had none of these powers.

Although most high school students have given up smoking cigarettes, teachers have noticed a rise in other unhealthy practices. In response to their concerns, Congress recently has passed three laws. Law (1) is a tax on transporting marijuana into public schools. Law (2) is a tax on the sale of highly caffeinated energy drinks favored by high school students. Law (3) is an act requiring withholding 3% of federal funding to any school that serves high-caloric desserts with school lunches. None of these laws contains a "jurisdictional element" that would tie the regulated items (i.e., marijuana, energy drinks, and desserts) to interstate commerce in any way. Congress also made no findings regarding any possible effects on interstate commerce. Congress expects to raise only minimal taxes and not to save any federal funding. Instead, the laws are designed to affect student behavior. Which law, if any, is very likely to be declared unconstitutional? A Law (1) because it violates the Penalty Doctrine. B Law (2) because Congress cannot impose taxes on intrastate transactions. C Law (3) because Congress could not regulate the serving of high-caloric desserts under the Commerce Clause unless the activity somehow affected interstate commerce. D None of these laws.

D None of these laws. Correct. None of these laws is likely to be declared unconstitutional.

The federal government recently purchased a large U.S.-based airplane manufacturer that was facing bankruptcy. Government officials explained that they were concerned that if this key manufacturer failed, domestic capacity to build large aircraft would be greatly diminished, seriously threatening national security. The United States now seeks to make the newly acquired manufacturer more profitable. Which legislative measure is most likely not permitted under the Commerce Clause? A Prohibiting U.S. air carriers from purchasing foreign-made aircraft for a one-year period. B Exempting the manufacturer from expensive federal labor and safety regulations. C Limiting the number of aircraft competing manufacturers can produce. D Requiring major corporations to purchase a specified number of corporate jets from the manufacturer even if they do not want to purchase any aircraft from any manufacturer.

D Requiring major corporations to purchase a specified number of corporate jets from the manufacturer even if they do not want to purchase any aircraft from any manufacturer. Correct. The Supreme Court held in National Federation of Independent Business v. Sebelius (2012) that Congress cannot compel "individuals not engaged in commerce to purchase an unwanted product." Although the case concerned individuals who did not want to buy health insurance, this holding probably also prevents Congress from requiring corporations to make unwanted purchases in areas of commerce in which they are not currently active.

After hearing news of several horrific attacks on recent immigrants, the President made, and the Senate approved, a new treaty with Mexico. The treaty required each country to create a claim and authorize a civil action for victims of violence motivated by immigration status. Congress immediately passed such a law implementing this requirement. The first defendant sued under the law claims that it is unconstitutional under United States v. Morrison (2000), which held that the Commerce Clause did not give Congress power to enact a nearly identically worded statute creating a civil action for gender-motivated violence. Which of the following arguments for the plaintiff is the strongest? A Morrison is distinguishable because crimes of violence that are motivated by immigration status are a form of economic activity. B The Supreme Court effectively overruled Morrison in Gonzalez v. Raich (2005). C The plaintiff can recover directly under the treaty with Mexico and need not rely on the implementing law. D The treaty power authorizes Congress to regulate matters that might otherwise be beyond the scope of federal power.

D The treaty power authorizes Congress to regulate matters that might otherwise be beyond the scope of federal power. Correct. The Supreme Court held in Missouri v. Holland (1920) that the "treaty power," which derives from the authority of the President to make treaties subject to the consent of two-thirds of the Senate and from treaties' status under the Supremacy Clause, authorizes Congress to regulate matters that might otherwise be beyond the scope of federal power. The scope of this power remains uncertain, see Bond v. United States (2014), but of the various choices this is the best argument.

Jane Dunston, the widowed mother of a four-year-old child, was convicted of assault and burglary and sentenced to five years in prison. After her conviction, the state Department of Social Services petitioned a state trial court to terminate Dunston's parental rights. The termination of parental rights would result in Dunston's child being transferred to the custody of her grandparents or to foster care. After receiving notice of the hearing at which the court would decide whether to terminate her parental rights, and the opportunity to appear personally at the hearing, Dunston, who is indigent and unable to afford a lawyer, filed a motion asserting that the Due Process Clause requires the state to appoint a lawyer to represent her at the hearing.Should the court grant Dunston's motion? A No, because Dunston's conviction deprived her of any legitimate claim of entitlement to continued parental rights. B No, because the court has provided notice and an opportunity for Dunston to appear in person before deciding whether to terminate Dunston's parental rights, and the Due Process Clause is always satisfied when the state provides notice and the opportunity to be heard in person. C Yes, because Dunston has a constitutionally protected liberty interest in directing the upbringing of her child, and deprivation of that liberty would constitute a grievous loss to Dunston. D Yes, if after balancing Dunston's interest in maintaining her parental rights, the risk of an erroneous deprivation without appointed counsel, and the government's interest, the court concludes that appointed counsel is warranted.

D Yes, if after balancing Dunston's interest in maintaining her parental rights, the risk of an erroneous deprivation without appointed counsel, and the government's interest, the court concludes that appointed counsel is warranted. Correct. Terminating parental rights would deprive Dunston of a liberty interest, and the test for what process is due before such a deprivation is the balancing test in Mathews v. Eldridge (1976).


Ensembles d'études connexes

NBDE II Prothodontics: General Considerations (impression materials, occlusion, lab)

View Set

Organization | Employee and Labor Relations - SHRM-CP

View Set

Principles of Management Exam 2 part 3

View Set

Bio Lab Exam #1 (Chemical Composition Q's)

View Set

FOCUSPOINT INN PERSON EXAMPLES OF THINGS DONE

View Set